Respiratory Unit 3

अब Quizwiz के साथ अपने होमवर्क और परीक्षाओं को एस करें!

Health care-associated pneumonia is defined as pneumonia diagnosed in a patient who was hospitalized within what timeframe? A. Last 30 days B. Last 48 days C. Last 60 hours D. Last 72 hours E. Last 90 days

The correct answer is 90 days (E). Health care-associated pneumonia is the diagnosis when a patient with pneumonia has been hospitalized within the last 90 days. These patients are at higher risk of drug-resistant pathogens such as Pseudomonas, MRSA, and other multidrug-resistant organisms. The other choices, 30 days (A), 48 days (B), 60 hours (C), and 72 hours (D), are not correct.

A 45-year-old businessman sees his primary care doctor. He returned from a trip from China 2 days ago. He said he developed a "cramp" in his left calf during the flight and now has minor swelling in his left foot. He is concerned because his daily run caused discomfort in his calf. His vital signs are normal, and his exam is significant for minor pitting edema around his ankle. His left calf is very tender to deep palpation. Which diagnostic test should be ordered first? A. Complete blood count B. Compression ultrasonography C. CT scan with contrast of lower extremities D. D-dimer assay E. No further work-up is needed. He should follow up in a week if his symptoms have not resolved.

The correct answer choice is D-dimer assay (D) because this patient is moderate risk for DVT. If the assay comes back low, then no further work-up is necessary. If the D-dimer is elevated, then compression ultrasonography (B) should be performed on the affected limb. DVT should be treated as early as possible to improve outcomes, so follow-up without a work-up is inappropriate (E). A CT scan with contrast (C) is unnecessarily expensive and invasive when compared with ultrasonography. A complete blood count (A) is also unnecessary because there is little concern for infection, anemia, or thrombocytopenia.

A 55-year-old woman with end-stage renal disease presents to her nephrologist to discuss options for dialysis. She has become increasingly nauseated in recent weeks. On exam, she appears calm but is noted to be tachypneic to rate of 24/min with a blood pressure of 158/92 mm Hg. Her lungs are clear and the heart is normal. Her blood pH is 7.25. What is the most likely cause of her elevated respiratory rate? A. Anxiety B. Hyperkalemia C. Hypoxia D. Metabolic acidosis E. Pericarditis

The correct answer choice is metabolic acidosis (D). Because of their kidney dysfunction, patients with end-stage renal disease often develop a chronic metabolic acidosis. To compensate for the increased acid burden, patients will breathe more quickly to "blow off" the extra CO2 and increase their pH back towards normal. So the pH is low, the HCO3 is low, and the pCO2 is low in this condition. Anxiety (A) and hypoxia (C) would lead to a respiratory alkalosis with high pH. Hyperkalemia (B) is seen in chronic kidney disease and causes cardiac arrhythmias but not hyperventilation. Pericarditis (E) causes chest pain and might cause a respiratory alkalosis, so the pH would be high. There are no exam findings of pericarditis (pericardial friction rub) or complaints of chest pain.

A 38-year-old HIV-positive woman is admitted to the hospital with severe respiratory distress, malaise, fever, and recent weight loss. Chest x-ray shows diffuse, bilateral infiltrates in the lower lung fields. Sputum cultures return revealing growth of Mycobacterium avium complex (MAC). Below what CD4+ T-cell count are patients at high risk for this infection? A. 200 cells/µL B. 100 cells/µL C. 50 cells/µL D. 25 cells/µL E. 10 cells/µL

The correct answer is 50 cells/µL (C). HIV patients who have a CD4+ T cell count below 50 cells/µL are at high risk for developing MAC infection. Counts below 200 cells/µL (A) and 100 cells/µL (B) are not correct because these patients, though immunocompromised, are not known to be at particularly high risk for MAC. Counts below 25 cells/µL (D) and 10 cells/µL (E) are much lower than the cutoff of 50 cells/µL and are therefore incorrect as well.

A 33-year-old professional mountain guide presents to the outpatient clinic to consult about her trip into the Patagonia mountain range in South America. She will spend several weeks at high altitude and is concerned about her ability to lead expeditions while having to rapidly adjust to the elevation. The physician prescribes her acetazolamide, a drug that inhibits carbonic anhydrase and leads to renal excretion of bicarbonate buffer. What acid-base disturbance will this drug cause, and how will it help her adjust to the high altitude? A. Metabolic acidosis: causes compensatory hyperventilation B. Metabolic alkalosis: decreases RBC oxygen binding C. Respiratory acidosis: decreases RBC oxygen release D. Respiratory acidosis: increases blood pCO2prior to arriving E. Respiratory alkalosis: normalizes blood pH

The correct answer choice is metabolic acidosis, which causes compensatory hyperventilation (A). At higher altitudes, there is a lower concentration of oxygen in the air, so we breathe faster (respiratory alkalosis) to meet our oxygen needs. This can cause headaches and nausea (altitude sickness). By inhibiting carbonic anhydrase, we increase renal bicarbonate excretion and this induces a metabolic acidosis. This causes a compensatory hyperventilation, so when she arrives at altitude, she will breathe faster and take in more oxygen. It will also normalize her pH, reducing symptoms. The other choices do not accurately describe the effect of acetazolamide nor present logical benefits. Lowering the serum bicarbonate level with acetazolamide would cause metabolic acidosis, not alkalosis (B) or respiratory acidosis (C and D) or respiratory alkalosis (E). Note that in both metabolic acidosis and respiratory alkalosis, the blood pCO2 drops, but in the first case it is a compensation, and in the second it is a primary hyperventilation. Regarding metabolic alkalosis (B) and respiratory acidosis (C), we would like more oxygen released to tissues in hypoxic environments, not less. Acidosis improves oxygen release to tissues, not decreases it (C). That is another theoretical advantage of acetazolamide, as the induced metabolic acidosis will improve oxygen release to tissues.

Which of the following can be used specifically for atopic asthma due to its IgE binding effect? A. Albuterol B. Corticosteroids C. Ipratropium bromide D. Montelukast E. Omalizumab

The correct answer choice is omalizumab (E). Omalizumab is a monoclonal antibody against IgE. It prevents the generation of type 1 hypersensitivity on exposure to an allergen. Albuterol (A) is a short-acting β-agonist. Corticosteroids (B) work by suppressing the inflammatory response. Ipratropium bromide (C) is a cholinergic receptor antagonist. Montelukast (D) is a leukotriene receptor inhibitor.

A 63-year-old former Navy aviator presents to the outpatient clinic with a chief complaint of increased work of breathing. He describes increased dizziness and difficulty catching his breath over the past 6 months but denies orthopnea or lower extremity edema. He has had a recent bout of gastroenteritis, with vomiting and diarrhea, after eating at a new restaurant. On exam, he appears anxious. There are occasional crackles in his lungs. His arterial blood gas shows: pH 7.55 pCO2 25 mm Hg HCO3 21 mEq/L pO2 70 mm Hg O2 saturation 85% Which of the following choices best describes his acid-base status and underlying pathophysiology? A. Combined metabolic and respiratory alkalosis B. Metabolic alkalosis due to chronic diarrhea C. Metabolic alkalosis due to vomiting D. Respiratory alkalosis due to anxiety E. Respiratory alkalosis due to hypoxia

The correct answer choice is respiratory alkalosis due to hypoxia (E). The patient's ABG shows elevated pH and decreased pCO2, consistent with a respiratory alkalosis. His reduced pO2 and oxygen saturation suggest a hypoxic stimulus for the respiratory alkalosis, likely due to lung disease (note the crackles on lung exam). In metabolic alkalosis (B and C) his pCO2 would be increased. Respiratory alkalosis due to anxiety (D) would demonstrate normal lungs and a normal pO2. A combined (mixed) disorder (A) is unlikely, as it usually leads to a normal pH or an unusual pattern of changes in pH, pCO2, and HCO3.

A 35-year-old African-American woman presents to the clinic to be evaluated for difficulty in inspiration. The doctor suspects sarcoidosis, a restrictive lung disease. What FEV1/FVC ratio is likely to be determined by spirometry? A. 0.60 B. 0.64 C. 0.66 D. 0.68 E. 0.85

The correct answer is 0.85 (E). Restrictive lung diseases have a FEV1/FVC ratio of 0.8 or more. Normal lungs have a FEV1 ratio of about 0.80. And obstructive lung diseases have a ratio of below 0.7. Therefore, the other choices, 0.60 (A), 0.64 (B), 0.66 (C), and 0.68 (D), refer to obstructive lung disease rather than restrictive lung disease.

A 72-year-old man presents to the emergency department with a 1-day history of shortness of breath, productive cough, and chest pain that is worse on deep inspiration. He is conscious, alert, and oriented. His temperature is 38.8°C (101.84°F), pulse rate 120/min, blood pressure 110/78 mm Hg, respiratory rate 28/min, and oxygen saturation 95% on room air. Laboratory testing revealed urea of 7.1 mmol/L. What is his CURB-65 score? 0 1 2 3 4

The correct answer is 2 (C). CURB-65 is a valuable tool that can be used to stratify the severity of pneumonia and guide its management. It measures severity of illness based on 5 factors: confusion, urea >7 mmol/L, respiratory rate >30/min, blood pressure (systolic <90 mm Hg or diastolic <60 mm Hg), and age >65 years. This patient scores 2 on the CURB-65 scoring system because his age is older than 65 years and his urea is above 7 mmol/L. The other parameters are below the CURB-65 threshold. Based on this score, this patient should be admitted and treated with IV antibiotics as an inpatient. Score 0 is wrong because this patient meets two CURB-65 criteria (A). Score 1 is also wrong for the same reason (B). Score 3 (D) and score 4 (E) are incorrect because this patient's blood pressure, respiratory rate, and mental status are unimpaired.

A 39-year-old patient has been admitted to your inpatient service. The following morning, you check on the lab results and find that the QuantiFERON-Gold assay has returned positive for tuberculosis. Which of the following treatment regimens is considered first-line therapy for tuberculosis? A. One month of isoniazid, pyrazinamide, and ethambutol followed by 6 months of rifampin B. One month of isoniazid, pyrazinamide, ethambutol, and rifampin followed by 6 months of isoniazid and rifampin C. Two months of isoniazid and rifampin followed by 4 months of isoniazid, pyrazinamide, ethambutol, and rifampin D. Two months of isoniazid, pyrazinamide, ethambutol, and rifampin followed by 4 months of isoniazid and rifampin E. Three months of isoniazid, pyrazinamide, and ethambutol followed by 6 months of isoniazid and rifampin

The correct answer is 2 months of isoniazid, pyrazinamide, ethambutol, and rifampin followed by 4 months of isoniazid and rifampin (D). One month of initial treatment followed by 6 months (A, B) is incorrect as is 3 months of treatment followed by 6 months (E) because each of these choices states the incorrect duration of time for both the initial phase and continuation phase. Furthermore, incorrect combination therapies are listed for both phases in two of the choices (A, E). Because the appropriate initial phase and continuation phase combo therapies are switched (C), this choice is incorrect. medication

A 52-year-old woman with a history of COPD (not receiving home oxygen), hypertension, and diabetes mellitus presents to the emergency department with a 2-day history of shortness of breath. She is a current smoker with a half-pack per day usage. Her vital signs are notable for a respiratory rate of 33/min. Breath sounds are absent over the left apical lung fields. What is the most likely underlying mechanism for the development of her pathology? A. Apical bleb rupture B. Barotrauma C. COPD D. Pneumonia E. Trauma

The correct answer is COPD (C). In this patient, who has a left-sided pneumothorax, the most likely cause is her underlying lung disease. COPD is the most common cause of a secondary spontaneous pneumothorax. Apical bleb rupture (A) is the most common cause of a primary spontaneous pneumothorax but is less likely in this patient who has COPD. Barotrauma (B) and trauma (E) are also less likely in this situation. There is no reason she would have been mechanically ventilated (B), and there is no report of trauma (E). This patient does not have signs (eg, crepitation on chest auscultation) and symptoms (eg, cough) suggestive of pneumonia (D).

After spirometry is performed, the flow-volume loop shows a decreased flow in the effort-dependent segment of expiration. What value does this refer to? A. FEV1 B. FVC C. RV D. TLC E. TV

The correct answer is FEV1 (A). The initial fast increase of air is effort dependent as it is forcing the air out of the larger airways. In obstructive lung diseases, the effort-dependent segment flow rate is decreased because airways can collapse due to pressure differentials. As the effort-dependent segment occurs at the beginning of expiration, it would affect the volume expired in the first second, FEV1. FVC (B), RV (C), TLC (D), and TV (E) may be changed in obstructive lung disease but should not be decreased.

Which spirometry value most likely reflects an obstructive lung disease, such as chronic bronchitis? A. FEV1/FVC ratio of less than 0.7 B. FEV1/FVC ratio of more than 0.7 C. High FEV1 D. High peak expiratory flow (PEF) E. Low total lung capacity (TLC)

The correct answer is FEV1/FVC ratio of less than 0.7 (A). In obstructive lung disease, one would expect the FEV1, PEF, and FEV1/FVC ratio to decrease. In addition, one would expect the TLC to increase because of air trapping in the lungs. So an FEV1/FVC ratio of more than 0.7 (B), high FEV1 (C), high PEF (D), and low TLC (E) are incorrect representations of chronic bronchitis.

Which of the following cytokines is chemotactic for eosinophils in atopic asthma? A. IL-2 B. IL-4 C. IL-5 D. IL-9 E. IL-13

The correct answer is IL-5 (C). IL-5 is implicated in atopic asthma and is responsible for eosinophilic chemotaxis. IL-2 (A) is a Th1-associated cytokine and not known to be associated with asthma. IL-4, IL-9, and IL-13 (B, D, and E) are Th2 cytokines and linked to asthma but are not chemotactic for eosinophils. IL-4 increases IgE, IL-9 increases the Th2 hypersensitization, and IL-13 increases mucus production. medication

Which of the following microbes uses a macrophage infectivity potentiator protein as a virulence factor? A. Chlamydophila pneumoniae B.Legionella pneumophila C. Mycoplasma pneumoniae D. Staphylococcus aureus E. Streptococcus pneumoniae

The correct answer is Legionella pneumophila (B). Virulence factors are tools used by microbes to help cause infection. L pneumophila uses a macrophage infectivity potentiator (Mip) protein and type IV secretion system (Dot/Icm), which help it invade macrophages and proliferate within them. Chlamydophila pneumoniae uses a type III system to inject proteins into their hosts (A). Mycoplasma pneumoniae produces toxic substances that can kill cells such as superoxide radicals and hydrogen peroxide (C). Staphylococcus aureus produces α-hemolysin, which is a pore forming cytotoxin (D). Streptococcus pneumoniae has a special polysaccharide capsule that protects it from phagocytosis and destruction by host macrophages (E).

Which of the following pathogens is a common cause of an atypical presentation of pneumonia? A. Haemophilusinfluenzae B. Mycobacterium tuberculosis C. Mycoplasmapneumoniae D. Staphylococcusaureus E. Streptococcuspneumoniae

The correct answer is Mycoplasma pneumoniae (C). Mycoplasma is a common cause of atypical pneumonia, while Haemophilus influenzae (A), Staphylococcus aureus (D), and Streptococcus pneumoniae (E) are all common causes of typical pneumonia. Mycobacterium tuberculosis (B) can cause pneumonia but typically presents with fever, chills, cough, weight loss, night sweats, and hemoptysis.

A patient with a history of dementia is hospitalized for a pulmonary abscess that contains anaerobic, gram-positive cocci. What is the etiologic pathogen? A. Bacteroides fragilis B. Fusobacteriumnecrophorum C. Klebsiella pneumoniae D. Nocardia E. Peptostreptococcus

The correct answer is Peptostreptococcus (E). Bacteroides fragilis (A) and Fusobacterium necrophorum (B) are anaerobic, gram-negative rods. Klebsiella pneumoniae (C) is an aerobic, gram-negative rod. Nocardia (D) is aerobic, with weakly acid-fast branching filaments.

Which organism commonly causes infective exacerbations of bronchiectasis? A. Borrelia burgdorferi B. Clostridium difficile C. Escherichia coli D. Pseudomonas spp E. Streptococcus pyogenes

The correct answer is Pseudomonas spp (D). One of the most common causes of infective exacerbations of both bronchiectasis and cystic fibrosis is Pseudomonas. Borrelia burgdorferi (A) are spirochete bacteria that cause Lyme disease. Clostridium difficile (B) are anaerobic bacteria that cause pseudomembranous colitis. Escherichia coli (C) are the most common cause of urinary tract infections, although they can cause many other enteric infections as well. Streptococcus pyogenes (E) causes the classic strep throat.

What is the name of the test used to diagnose newborns with CF after a positive screening test? A. IRT B. Methacholine challenge C. OGTT D. TSH E. QPIT

The correct answer is QPIT (E). IRT (immunoreactive trypsinogen) (A) is a commonly used screening test for CF; it is not diagnostic because it can be positive in conditions other than CF. A sweat test (QPIT) must be done after a newborn screening test is positive to diagnose or rule out CF. The methacholine challenge (B) is a test used to diagnose asthma. OGTT (C) is one of the tests used to diagnose diabetes mellitus. TSH (D) is a hormone in your body used to stimulate the production of thyroid hormones.

A 70-year-old man has recently been diagnosed with IPF after several months of hemoptysis and dyspnea. Which of the following medications has been demonstrated to confer a clinical benefit to patients with IPF? A. Amiodarone B. Colchicine C. Pirfenidone D. Salmeterol E. Theophylline

The correct answer is pirfenidone (C). Pirfenidone is an antifibrotic agent that inhibits collagen synthesis. It is one of two agents (the other is nintedanib) that provide a clinical benefit to patients with IPF. Amiodarone (A) is an antiarrhythmic that does not help in IPF and is actually known to cause a secondary pulmonary fibrosis. Colchicine (B), salmeterol (D), and theophylline (E) have not demonstrated a clinical benefit in patients with IPF.

A patient comes to the emergency department with a 2-day history of fevers, chills, cough with green sputum, dyspnea, and significant malaise. A dense left lower lobe consolidation is noted on chest x-ray. Which of the following pathogens is most likely to present in this manner? A. Chlamydophilapneumoniae B. Legionellapneumophila C. Mycoplasmapneumoniae D. Streptococcuspneumoniae E. Staphylococcusepidermidis

The correct answer is Streptococcus pneumoniae (D). The patient is presenting with a typical pneumonia, which is characterized by acute onset, fevers, dyspnea, malaise, and productive cough. In addition, the chest x-ray shows a consolidation, which is consistent with a typical pneumonia. Chlamydophila pneumoniae (A), Legionella (B), and Mycoplasma pneumoniae (C) are all causes of atypical pneumonia. Staphylococcus epidermidis (E) causes many different infections but does not cause pneumonia, unlike its family member Staphylococcus aureus.

Which of the following is implicated in the pathogenesis of idiopathic pulmonary fibrosis? Bleomycin A. IL-2 B. IL-6 C. TGF-β D. TNF

The correct answer is TGF-β (D). Pulmonary fibrosis caused by bleomycin (A) would be classified as secondary pulmonary fibrosis. IL-2 (B), IL-6 (C), and TNF (E) are not implicated in the pathogenesis of IPF at this time.

A 7-year-old boy who shows signs of cyanosis is brought to the emergency room by his mother. The boy began having difficulty breathing after playing with a neighbor's new kitten. His mother says that this had happened before when the patient was around cats, but not to this extent, and had quickly self-resolved. The patient is given albuterol and his condition improves. Spirometry with bronchoprovocation testing is scheduled for the next day. What value after methacholine administration is an indicator of bronchial asthma? A. 20% decrease in FVC B. 20% decrease in FEV1 C. FEV1/FVC ratio above 0.8 D. Oxygen saturation below 95%

The correct answer is a 20% decrease in FEV1 (B) in relation to the spirometry prior to methacholine administration. This decrease in FEV1 is due to the constriction of the airways, causing obstruction and limiting the flow of expiration. A 20% decrease in FVC (A) is not typically associated with asthma or bronchoprovocation testing. Because asthma is an obstructive disease, an FEV1/FVC ratio above 0.8 would not indicate bronchial asthma (C). A drop in O2 saturation may be seen (D), but this is not the indicator determined in bronchoprovocation testing. medication

A 22-year-old 6'6" and 175-lb man presents to the emergency department with sudden-onset chest pain and shortness of breath. He has pain on only the right side of his chest. He has no significant medical history and no surgical history. He takes no medications. He reports he was in his usual state of health until this occurred. The man is a marathon runner and eats a well-balanced diet. He does not smoke or use any drugs. He has never experienced anything like this before. He is afebrile, his blood pressure 132/84 mm Hg on his left and 131/83 mm Hg on his right, pulse 88/min, and respiration rate 25/min. Auscultation reveals absent breath sounds on the right. What is the most likely mechanism underlying this man's pathology? A. Accumulation of air within the pleural space B. Accumulation of fluid within the pericardium C. Blockage of the pulmonary vessels due to a thrombus D. Intimal tear of the aorta E. Ischemia of the myocardium

The correct answer is accumulation of air within the pleural space (A). This man most likely has a pneumothorax. His tall, thin stature is characteristic of those likely to suffer a pneumothorax. Further, without a significant medical history or risk factors for other pathology, this is the most likely choice. Cardiac tamponade—accumulation of fluid within the pericardium (B)—likely would occur in the setting of a recent illness and would likely be associated with hypotension and pulsus paradoxus. There is no indication or risk factor for this man to experience a pulmonary embolism—blockage of the pulmonary vessels due to a thrombus (C). Without drug use, genetic predisposition, or hypertension, it is unlikely that this man would be experiencing an aortic dissection, or intimal tear of the aorta (D). Given the age, diet, and functional status of this individual, it is highly unlikely he is having a myocardial infarction, or ischemia of the myocardium (E).

A 49-year-old homeless man arrives at the emergency room complaining of productive cough with streaks of blood. He reports that he has had this cough for more than 3 months with associated fever, night sweats, and weight loss. Vitals reveal a mild fever of 101°F, respiration rate 24/min, heart rate 98/min, and blood pressure 130/86 mm Hg. His oxygen saturation is 93%. Which of the following tests will help confirm the most likely diagnosis? A. Chest x-ray B. Gram stain of sputum cultures C. Acid-fast bacillus smear of sputum and PCR test D. Serologic test E. Tuberculin skin test

The correct answer is acid-fast bacillus smear of sputum and polymerase chain reaction (PCR) test (C). The best diagnostic confirmatory tests are culture of the organism (the gold standard, which allows drug sensitivity testing but takes 2-3 weeks), DNA amplification (rapid), or microscopy using acid-fast staining. Though chest radiographs (A) can help evaluate tuberculosis patients, some may not have any obvious lesions on imaging. Gram stain (B) does not work well on mycobacteria because their cell walls are rich with mycolic acids, making them resistant to Gram staining. Serologies (D) are of little use in diagnosing tuberculosis because of the low sensitivity and specificity of these tests; humoral response is nonevident in tuberculosis (an intracellular infection, controlled by cell-mediated immunity). The tuberculin skin test (E) is only helpful in screening for reactivity to tuberculin protein antigens. It does not confirm diagnosis and cannot distinguish between active and latent infection.

What is the main virulence factor of Mycoplasma pneumoniae? A. Adhesins B. α-Hemolytic cytotoxin C. Multidrug-resistant genes D. Peroxide andfree radical production E. Polysaccharide capsule

The correct answer is adhesins (A). The main virulence factor of M pneumoniae is the adhesins at the tip of the organism that allow it to adhere to respiratory epithelial cells and cause infection. Though M pneumoniae can produce cytotoxic substances such as peroxide and free radicals (D), this is not its main characteristic that helps it infect the human host. Polysaccharide capsule (E) is the main virulence factor for S pneumoniae, α-hemolytic toxin (B) is the main virulence factor for S aureus, and multidrug-resistance (C) is a hallmark for enterococci bacteria.

What is the suggestive finding on chest x-ray of a pulmonary abscess? A. Air-fluid levels B. Bronchopneumonia C. Calcifications D. Interstitial infiltrates E. Lobar consolidation

The correct answer is air-fluid levels (A). Bronchopneumonia (B), interstitial infiltrates (D), and lobar consolidation (E) can be found in an uncomplicated pneumonia. Calcifications (C) can be suggestive of previous Mycobacterium tuberculosis infection or histoplasmosis.

A 28-year-old nonsmoking man with a 3-year history of emphysema returns to his doctor for an appointment to monitor his condition. What structure and location are most impacted by his emphysema? A. Alveolar septa, centriacinar B. Alveolar septa, panacinar C. Pulmonary capillaries, centriacinar D. Pulmonary capillaries, panacinar E. Respiratory bronchioles, lower lobe

The correct answer is alveolar septa, panacinar (B). Emphysema causes destruction of the alveolar septa. This patient's young age and lack of smoking history with a diagnosis of emphysema make it likely that he has A1AT deficiency. A1AT deficiency has a panacinar distribution due to the widespread absence of A1AT, rather than smoking-related centriacinar (A and C). Pulmonary capillaries (D) are affected only very late in the disease process. Respiratory bronchioles (E) can have decreased elasticity in emphysema, but not to the same impact as seen in the septa.

Which of the following substances are produced by macrophages in noncaseating granulomas? A. Angiotensin converting enzyme B. Calcitonin C. Interferon γ D. Interleukin 5 E. Thyroid stimulating hormone

The correct answer is angiotensin converting enzyme (A). Activated macrophages produce ACE, and measuring a serum ACE level is part of the work-up for sarcoidosis. Calcitonin (B) is produced by the parafollicular cells of the thyroid gland. IFN-γ (C) is produced by the CD4+ T cells that aid in the formation of granulomas, not the macrophages. IL-5 (D) is produced by Th2 T cells. Thyroid stimulating hormone (E) is produced by the anterior pituitary gland.

Every year, researchers develop new flu vaccines because the viruses mutate rapidly, producing new dominant strains. Which of the following correctly describe the genetic variability that rapidly occurs in influenza viruses? A. Antigenic shift is a nonsense mutation and antigenic drift is from a point mutation. B. Antigenic shift is from codon deletion and antigenic drift is from silent mutation. C. Antigenic shift is from point mutations and antigenic drift is from segment rearrangements. D. Antigenic shift is from segment rearrangements and antigenic drift is from point mutations. E. Antigenic shift is from silent mutation and antigenic drift is from codon deletion.

The correct answer is antigenic shift is from rearrangement of viral genome segments and antigenic drift is from point mutations (D). Although point mutation is associated with antigenic drift, a nonsense mutation does not cause antigenic shift (A). Silent mutation and codon deletion are not associated with antigenic shift or drift (B). Point mutation causes antigenic drift but not shift, and segment rearrangements cause antigenic shift but not drift (C). Further silent mutation does not lead to antigenic shift, and codon deletion does not lead to antigenic drift (E).

An 80-year-old man who is a retired plumber presents to the office with a severe cough, shortness of breath at rest, and hemoptysis (blood-tinged sputum). Chest imaging reveals fibrosis of both lower lung lobes and pleural plaques. What is the most likely diagnosis? A. Asbestosis B. Coal workers' pneumoconiosis C. Pneumonia D. Silicosis E. Tularemia

The correct answer is asbestosis (A). The pulmonary symptoms combined with a history of working in the plumbing industry point toward asbestosis. In addition, damage to the lower lung lobes and pleural plaques are characteristics of asbestosis. Coal workers' pneumoconiosis (B) usually affects individuals who worked in the coal mining industry. Pneumonia (C) can cause shortness of breath, cough, and hemoptysis; however, it would not be expected to manifest as fibrosis on chest imaging. Silicosis (D) usually affects individuals who worked in mines or foundries; in addition, the lower lung lobes are not usually affected. Tularemia (E) is an infection that is often caused by exposure to rabbits and may cause pulmonary manifestations.

Which of the following is an objective finding for right heart failure? A. Ascites B. Orthopnea C. Paroxysmal nocturnal dyspnea D. Pulmonary edema E. Systolic ejection murmur at the right second intercostal space

The correct answer is ascites (A). Left untreated, right ventricular hypertrophy can result in right heart failure. When the right side of the heart fails to pump effectively, there is more fluid left over in the heart after each beat. Subsequent fluid that returns to the right atrium gets backed up like a traffic jam. This leads to signs consistent with the systemic fluid overload seen in right heart failure, including jugular venous distention, ascites, and peripheral edema. Orthopnea (B), paroxysmal nocturnal dyspnea (C), and pulmonary edema (D) are findings more consistent with left-sided heart failure, not right-sided. Systolic ejection murmur at the right second intercostal space (E) is consistent with aortic stenosis.

A 7-year-old boy presents to the emergency room with shortness of breath and wheezing. It is determined that he is having exacerbation of his asthma. It is treated appropriately, and symptoms resolve. Which of the following is the most likely to be seen in this patient the next morning? A. Above normal FVC B. Below normal FEV1 C. Below normal FEV1with methacholine testing D. Below normal FeNO E. Normal FeNO

The correct answer is below normal FEV1 ratio with methacholine testing (C). Even when asthma patients are not experiencing symptoms, they are likely hyper-reactive to bronchoprovocation testing. Above normal FVC (A) and below normal FEV1 (B) are wrong because the patient would likely have normal FEV1 and FVC without bronchoprovocation testing. Below normal FeNO (D) and normal FeNO (E) are incorrect because the patient would likely have increased fractional exhalation of nitric oxide (hyper-reactive mucosa releases high levels of nitric oxide).

A 6-year-old boy with a medical history of hay fever presents to the emergency department with acute onset episode of dyspnea, wheezing, and cyanosis. What is the cause of his condition? A. Alveolar destruction B. Bronchial necrosis C. Bronchoconstriction D. Hilar lymphadenopathy E. Interstitial lung disease

The correct answer is bronchoconstriction (C). The child in this question has classic signs of asthma (hypersensitivity history, wheezing, and hypoxemia) which is caused by hypersensitivity-induced bronchoconstriction. Alveolar destruction (A) is associated with emphysema. Bronchial necrosis (B) is associated with bronchiectasis. Hilar lymphadenopathy (D) is associated with sarcoidosis. Interstitial lung disease (E) is less likely to cause wheezing.

When you evaluate a patient's pleural fluid, the fluid collection is determined to be most likely transudative in nature. Using Light criteria, which of the following lab values would support this assertion? A. Pleural fluid cholesterol level <250 B. Pleural fluid cholesterol level >250 C. Pleural fluid LDH/serum LDH ratio <0.6 D. Pleural fluid LDH/serum LDH ratio >0.6 E. Pleural fluid protein/serum protein ratio >0.5

The correct answer is pleural fluid LDH/serum LDH ratio <0.6 (C). This question requires recollection of the components of Light criteria. Pleural fluid cholesterol level <250 and >250 (A and B) are not a part of Light criteria. Pleural fluid LDH/serum LDH ratio >0.6 (D) and pleural fluid protein/serum protein ration >0.5 (E) are both values consistent with an exudate.

Five weeks after her 13-year-old daughter began taking a new asthma drug, a worried mother brought her to her primary care doctor because of some "new growth" in her mouth. On examination, there appears to be a white plaque coating her tongue. The plaque was easily scrapped off with a tongue depressor. Which of the following is most likely responsible for this patient's presentation? A. Albuterol B. Budesonide C. Ipratropium D. Miconazole E. Montelukast

The correct answer is budesonide (B). This 13-year-old girl is presenting with symptoms of oral candidiasis or oral thrush. Oral candidiasis can occur in people who are immunosuppressed. But when occurring in an otherwise healthy young patient, iatrogenic causes should be suspected. Budesonide belongs to the corticosteroid family, which can be used as long-term control therapy for asthma. One of the adverse effects of inhaled steroids is the development of oral candidiasis. Albuterol (A) is a β2 agonist and is not commonly associated with the development of oral candidiasis. Ipratropium (C) is a muscarinic antagonist and is associated with anticholinergic side effects but not oral candidiasis. Miconazole (D) is an anti-fungal that can treat oral candidiasis, but it is not used to treat asthma. Montelukast (E) is an anti-leukotriene with few side effects.

Which of the following would be seen on histologic examination of lung tissue from a patient with a history of coal workers' pneumoconiosis? A. Carbon-laden macrophages B. Hyaline membranes C. Iron-laden macrophages D. Squamous metaplasia E. Ferruginous bodies

The correct answer is carbon-laden macrophages (A). Prolonged exposure to coal leads to an inflammatory reaction mediated by macrophages, which ingest the coal. Hyaline membranes (B) are seen in acute respiratory distress syndrome. Iron-laden or hemosiderin-laden macrophages are seen in patients with congestive heart failure (C). Squamous metaplasia (D) is commonly seen in smokers. Ferruginous bodies (E) are seen in patients with asbestosis.

What is the most likely distribution of alveolar destruction of the patient described above? A. Centriacinar B. Irregular C. Panacinar D. Paraseptal E. Perihilar

The correct answer is centriacinar distribution (A). The patient's history of smoking makes it most likely to be centriacinar. Smoke is inhaled into the lung, moving its way through the airways. This causes a greater damaging effect to the area surrounding the respiratory bronchiole. Irregular emphysema (B) is associated with fibrosis rather than smoking. Panacinar emphysema (C) is associated with A1AT deficiency, not smoking. Paraseptal emphysema (D) is associated with smoking and fibrosis but is not as likely a diagnosis, because centriacinar emphysema is much more prevalent. Perihilar distribution (E) of granulomas is seen in several diseases, such as tuberculosis, sarcoidosis, or infections.

A 37-year-old man is rushed to the emergency department after falling from a ladder while cleaning his gutters. He was responsive in the field and has been conversant throughout transport. On exam, he has decreased breath sounds on the left lung fields. His blood pressure is 120/80 mm Hg, pulse 95/min, and saturation 98% on room air. What is the most appropriate next step in the management of this individual? A. Provide nasal cannula with 100% fraction of inspired oxygen (FiO2) B. Chest radiograph C. Needle thoracostomy D. Tube thoracostomy E. Insertion of a tracheal tube

The correct answer is chest radiograph (B). This man had a traumatic accident that caused decreased left lung breath sounds in the setting of normal vital signs. The initial step in evaluation should be getting a chest radiograph. It is possible that he has a tension pneumothorax, which would require immediate medical intervention via needle thoracostomy (C) or tube thoracostomy (D). But because he is stably breathing, evaluation using x-ray of what may be causing his decreased breath sounds is warranted before any procedures (B). Because he is breathing spontaneously at 98% oxygen saturation, there is no need for tracheal intubation (E) and no need for nasal cannula (A).

A 55-year-old man with diabetes mellitus and a 30-pack-year smoking history presents to the emergency department with worsening shortness of breath and vague pleuritic chest pain. He reports that he was in his usual state of health until the day before when he began to feel some pain in his left flank moving towards his shoulder. On exam, he appears in mild distress and is tachypneic. He has no jugular venous distention or stridor, and his cardiac exam reveals no murmurs, rubs, or gallops. Results of laboratory tests including chemistries, blood counts, and urinalysis are all within normal limits. An electrocardiogram (ECG) reveals no evidence of infarct or strain. What is the most appropriate first test? A. Chest radiograph B. Lower extremity duplex ultrasound exam C. Radionuclide scintigraphy D. Toxicology screen E. Transesophageal echocardiogram

The correct answer is chest radiography (A). This patient presents with a story of vague, nondescript chest pain suggestive of a pleural issue. After ruling out infarction with ECG, the next appropriate step would be to obtain imaging of the chest (A). A lower extremity duplex ultrasound exam (B) and radionuclide scintigraphy (C) would be appropriate options to evaluate for pulmonary embolism; however, neither is the first choice for imaging. A toxicology screen (D) would be of no benefit in a case like this. A transesophageal echocardiogram (TEE) (E) would offer evaluation of cardiac function; however, it would not be the first choice in evaluation of a patient presenting with these symptoms.

Of the following signs and symptoms, which one is most indicative of a diagnosis of bronchiectasis? A. Clubbing B. Chronic daily large-volume mucopurulent sputum production C. Dyspnea D. Frequent respiratory infections E. Hemoptysis

The correct answer is chronic daily large-volume mucopurulent sputum production (B). Although other respiratory diseases may cause a chronic productive cough, a presentation of prolonged chronic productive cough with copious mucopurulent sputum production should make you strongly consider a diagnosis of bronchiectasis. Clubbing (A), dyspnea (C), frequent respiratory infections (D), and hemoptysis (E) are all associated signs and symptoms of bronchiectasis, but they can also be associated with a multitude of other respiratory illnesses. They are not as specific for bronchiectasis as choice (B) is.

During normal quiet breathing, which of the following structures will demonstrate turbulent flow? A. Alveolar ducts B. Bronchi C. Bronchioles D. Terminal bronchioles E. Trachea

The correct answer is trachea (E). Turbulent flow occurs in larger airways with larger diameters because of increased flow velocity. During normal quiet breathing, turbulent flow occurs in the pharynx and trachea. Alveolar ducts (A), bronchi (B), bronchioles (C), and terminal bronchioles (D) are smaller airways than the trachea. Flow would be laminar in those parts of the respiratory tree. However, with forced inspiration and expiration, flow velocity increases, leading to turbulent flow even in smaller airways.

A 25-year-old man is wheeled into the emergency department because of a sudden loss of consciousness after a narcotic overdose. He has a temperature of 36°C, respiratory rate 6/min with adequate depth on both lungs, heart rate 120/min, and blood pressure 130/89 mm Hg. Arterial blood gas results are: pH 7.16 PaO2 59 mm Hg PaCO2 70 mm Hg What is the most likely cause of his high PaCO2? A. Airway obstruction B. Decreased dead space C. Decreased minute ventilation D. Increased CO2production E. Increased dead space

The correct answer is decreased minute ventilation (C). The patient's PaCO2 is too high (>40 mm Hg), which indicates that he has hypercapnia. Because his respiratory rate is 6/min, which is lower than normal, it indicates that he is hypoventilated. His minute ventilation is low due to low respiratory rate (VE = RR × VT). Airway obstruction (A) can cause low tidal volume, but because of adequate breathing depth, this is unlikely. Decreased dead space (B) will actually increase effective breathing and lower PaCO2. On the other hand, both increased CO2 production (D) and increased dead space (E) stimulate the respiratory rate, which is not observed in this patient.

A 28-year-old woman with a history of alcoholic cirrhosis presents to the hospital in acute respiratory distress. On examination, she is jaundiced, tachycardic, and tachypneic and her abdomen has a positive fluid wave. The chest x-ray shows bilateral pleural effusions with moderate volumes of fluid in both lungs. What is the most likely cause of her pleural fluid collection? A. Chest wall trauma B. Decreased interstitial oncotic pressure C. Decreased serum oncotic pressure D. Increased interstitial hydrostatic pressure E. Increased vascular permeability

The correct answer is decreased serum oncotic pressure (C). The patient in this vignette has alcoholic cirrhosis and ascites, as evidenced by her positive fluid wave. As a result of her ascites, it can be assumed that her liver synthetic function may be poor, thus leading to a hypoalbuminemic state. As a result, the intravascular oncotic pressure will be lower than interstitial pressure, leading to extravasation of fluids. This patient has no history of chest wall trauma (A). Decreased interstitial oncotic pressure (B), increased interstitial hydrostatic pressure (D), and increased vascular permeability (E) are all possible causes of pleural effusions but are less likely than (C) in a patient with cirrhosis.

A 19-year-old female patient presents to her college campus health care clinic complaining of pain and swelling in her right leg. She states that she has been stressed while studying for final exams, sitting in the library for long hours for the past few days preparing for an organic chemistry exam. She had some aching in her leg last night and woke up this morning with the swelling. She is alert and oriented and denies shortness of breath, chest pain, and abdominal pain. A physical exam is unremarkable other than severe swelling and erythema in her right leg distal to the mid-thigh, with large palpable superficial veins. Her vital signs are normal. She currently uses a contraceptive vaginal ring for birth control and is allergic to codeine. What diagnosis are you most concerned about? A. Baker cyst infection B. Compartment syndrome C. Deep vein thrombosis D. Lymphedema E. Pulmonary embolism

The correct answer is deep vein thrombosis (C). This patient's only complaint is right leg pain, redness, and swelling. In addition, she has spent large amount of time seated (stasis) over the past few days and is taking estrogen contraceptives (hypercoagulability). According to the Wells score, this patient is high risk and should be evaluated with compression ultrasonography for diagnosis. Pulmonary embolism (E) without respiratory symptoms is less likely. Without an antecedent event or injury, compartment syndrome (B) is much less likely. Baker cyst infections (A) should present more focally. Lymphedema (D) without antecedent damage to the lymphatic system is also unlikely and is not consistent with the acute presentation.

A 65-year-old man with a 40-pack-year history of smoking presents to the clinic due to a 1-year history of worsening shortness of breath and a 10-lb weight loss. On physical exam, the patient is seated in a tripod position with an increased chest diameter. Pulmonary function testing is performed. Among the findings, the FEV1/FVC ratio is 65%. Which of the following is the most likely cause of this disease? A. A1AT misfolding B. CFTRmutation C. Dynein arm dysfunction D. Elastase release from neutrophils E. Foreign body reaction

The correct answer is elastase release from neutrophils (D). The elastase cleaves the elastin in the septa of the alveoli, causing eventual destruction. Although α1-antitrypsin (A1AT) misfolding can cause emphysema (A), it is not as likely and would present at a younger age in combination with smoking. CFTR mutations (B), dynein arm dysfunction (C), and foreign body reactions (E) do not cause emphysema.

A 65-year-old woman presents with a 3-day history of cough, runny nose, shortness of breath, and chills. Her only medical history is a liver transplant 3 years ago. She has had no complications since then and states she otherwise has been healthy. Oxygen saturation is 93%. Which of the following would most suggest that the cause of her symptoms is viral rather than bacterial? A. Elevated monocyte count B. Elevated neutrophil count C. Radiographic imaging reveals left lower lobe consolidation D. Silver stain on sputum culture reveals causative organism E. Swollen nasal turbinates

The correct answer is elevated monocyte count (A). Of the choices, this is the clinical finding most aligned with a viral pneumonia. High neutrophils (B) are more indicative of bacterial invasion. Distinct consolidation on x-ray (C) is more often associated with bacterial pneumonia. Silver staining (D) is used for fungal organisms. Swollen turbinates (E) are a nonspecific finding for upper airway inflammation rather than pneumonia.

Which of the following conditions is not a restrictive lung disease? A. Pulmonary alveolar proteinosis B. Emphysema C. Hypersensitivity pneumonitis D. Idiopathic pulmonary fibrosis E. Sarcoidosis

The correct answer is emphysema (B). Pulmonary alveolar proteinosis (A), hypersensitivity pneumonitis (C), idiopathic pulmonary fibrosis (D), and sarcoidosis (E) are all types of restrictive lung disease.

A 34-year old patient comes to the physician after finding out her mother was recently diagnosed with idiopathic pulmonary fibrosis. She wants to know what the odds are that she might develop this condition. What should you tell her? A. She has a 25% chance of inheriting this condition. B. She has a 50% chance of inheriting this condition. C. She should be screened when she turns 50 years old with a low-dose CT scan. D. There is no evidence that IPF runs in families. E. Though there is a tendency for IPF to run in families, many cases are spontaneous.

The correct answer is that IPF tends to run in families, but many cases are spontaneous (E). There is no way to predict the patient's chances of developing IPF based on the fact that her mother developed it (A and B). There is currently no recommendation for screening in asymptomatic family members (C). Choice (D) is incorrect because genetic mutations have been established, although they do not account for all cases of IPF.

A 25-year-old woman presents to the office for a swollen, hot, and painful left calf. She is taking the pill for contraception but does not use any regular medication. She has never smoked. She drinks 5 units of alcohol a week. Her mother died of breast cancer. On examination, she has an erythematous, tender, warm left calf. Dorsiflexion of the foot produces pain at the calf. Which of the following increases her risk for venous thromboembolic disease? A. Age B. Alcohol use C. Estrogen contraceptive use D. Family history of breast cancer E. Smoking status

The correct answer is estrogen contraceptive use (C). Estrogen contraceptive use creates a hypercoagulable environment. Malignancy (eg, breast cancer) increases the risk of venous thromboembolic disease (VTE). However, we have not been told that the patient has a personal history of breast cancer (D). This patient is 25 years old, so her age does not increase risk of VTE (A). Her low-moderate alcohol use (B) does not increase her risk of VTE. Smoking is a major risk for VTE; however, the patient has never smoked (E), which decreases her risk.

Which of the following is not a possible source of a pulmonary embolism? A. Amniotic fluid B. Deep vein thrombosis C. Fat D. Fibrinolytic treatment E. Tumor

The correct answer is fibrinolytic treatment (D). Therapy with fibrinolytics would prevent clot and thrombosis formation. Amniotic fluid (A), deep vein thrombosis (B), fat (C), and tumors (E) are all possible causes of pulmonary emboli.

A 65-year-old white man comes to the clinic for assessment of previously diagnosed lung disease, and spirometry testing is performed. Which of the following lung volumes and capacities cannot be measured by spirometry? A. Expiratory reserve volume B. Functional residual capacity C. Inspiratory reserve capacity D. Tidal volume E. Vital capacity

The correct answer is functional residual capacity (B). The residual volume cannot be measured by spirometry because it is not part of expiration; therefore, the functional residual capacity cannot be determined, because it is calculated using the residual volume. Expiratory reserve volume (A), inspiratory reserve capacity (C), tidal volume (D), and vital capacities (E) can be determined, as they are not based on residual volume.

A 35-year-old woman complains of fever, weight loss, and productive cough. She has been homeless for several years and was recently released from jail. Because of the patient's risk factors, you need to rule out Mycobacterium tuberculosis. If you are the microbiologist working to culture and stain this organism, which of the following would you use? A. Growth at 37˚C on chocolate agar, stain with methylene blue B. Growth at 37˚C on Lowenstein-Jensen media, Ziehl-Neelsen staining C. Growth at 41˚C on blood agar, visualize with silver stain D. Growth at 41˚C on chocolate agar, visualize with Gram stain E. Growth at 41˚C on Lowenstein-Jensen media, visualize with Gram stain

The correct answer is growth on Lowenstein-Jensen media in 37˚C and visualized using Ziehl-Neelsen staining (B). Methylene blue (A) is not routinely used for visualization of M tuberculosis. The optimum growth temperature for M tuberculosis is 37°C, making (C), (D), and (E) automatically incorrect. Furthermore, blood agar (C) and chocolate agar (D) are not routinely used to culture this organism.

Which of the following proteins helps influenza attach to host cells so they can ultimately infect them? A. Hemagglutinin B. Immunoglobulin C. Neuraminidase D. Oseltamivir E. Reverse transcriptase

The correct answer is hemagglutinin (A). Immunoglobulins (B) help the immune system fight pathogens. Neuraminidase (C) allows the influenza virus to be released from an infected host cell and spread to other cells. Oseltamivir (D) is a neuraminidase inhibitor. Reverse transcriptase (E) transcribes RNA into DNA.

Which of the following is a possible treatment option for a patient presenting to the emergency department with a stable pulmonary embolus? A. Abciximab B. Heparin C. Mexiletine D. Propylthiouracil (PTU) E. Ranitidine

The correct answer is heparin (B), a drug that potentiates antithrombin and thus prevents clot formation and propagation. Abciximab (A) binds to the glycoprotein IIb/IIIa receptor on platelets and prevents aggregation but is indicated for unstable angina and percutaneous coronary intervention (and not PEs). Mexiletine (C) is a class IB antiarrhythmic used for acute ventricular arrhythmias (especially after myocardial infarction). Propylthiouracil (D) blocks the enzyme thyroid peroxidase, thus preventing synthesis of thyroid hormone. It is used to treat hyperthyroidism and would not be indicated in this patient. Ranitidine (E) is a histamine (H2) blocker used for the treatment of gastroesophageal reflux disease (GERD).

Which of the following parameters would most likely be seen in a patient presenting to the emergency department with an acute pulmonary embolus? A. Hypercapnia B. Hypocapnia C. Increased blood oxygen levels D. Increased cardiac output E. Metabolic alkalosis

The correct answer is hypocapnia (B). A patient with an acute pulmonary embolus is likely to experience tachypnea (increased respiratory rate), which would lower carbon dioxide levels. Hypercapnia (A) is incorrect, as a patient with an acute PE would usually be in a state of respiratory alkalosis and not acidosis. Increased blood oxygen levels (C) is incorrect because blood oxygen levels would more likely decrease. Increased cardiac output (D) is incorrect because cardiac output is more likely to drop in patients presenting with acute PE. Metabolic alkalosis (E) is incorrect, as an acute PE would usually not cause any major fluctuations in metabolic functions. The kidneys might compensate for the acute respiratory alkalosis (by excreting more bicarbonate), but this response takes days to occur.

Which of the following options is not considered part of the treatment of bronchiectasis? A. Antimicrobial agents B. Bronchial hygiene C. Immunotherapy D. Mucolytic agents E. Smoking cessation

The correct answer is immunotherapy (C). Immunotherapy, or immune-modulating therapy, uses antibody medications to treat autoimmune diseases. True bronchiectasis is not caused by an autoimmune process, so immunotherapies will not be helpful in treating it. Antimicrobial agents (A), bronchial hygiene (B), mucolytic agents (D), and smoking cessation (E) are all considered part of the treatment of bronchiectasis.

In patients with chronic bronchitis, what would be seen on lung biopsy? A. Deterioration of alveolar septa B. Increased bronchial mucosal layer C. Noncaseating granuloma D. Scar tissue E. Widespread inflammation

The correct answer is increased bronchial mucosal layer (B). The hypertrophy and hyperplasia of the bronchial mucus glands cause thickening of the layer. Deterioration of the alveolar septa (A) is seen in emphysema. Noncaseating granuloma is seen in sarcoidosis (C). Scar tissue is seen with some interstitial lung diseases like idiopathic pulmonary fibrosis (D). Widespread inflammation can be seen with hypersensitivity pneumonitis (E).

A mother brought her 12-year-old boy to the physician because she noticed that every time he plays soccer in the cold, he becomes short of breath. She explains that his symptoms are so severe that sometimes he has to leave the game, gasping for air. A diagnosis of exercise-induced asthma was made, and the boy was given a short-acting β2 agonist. What is the main mechanism of action of this medication? A. Adenylyl cyclase inhibition B. Decreased leukotriene production C. Increased cAMP production D. Inactivation of NF-κB E. Phosphodiesterase inhibition

The correct answer is increased cAMP production (C). Both short-acting and long-acting β2 agonists stimulate the β2 receptors found on the bronchial smooth muscle. Stimulation of the β2 receptor lead to the activation of adenylyl cyclase, which results in an increase of cAMP production. Increased cAMP then results in smooth muscle relaxation. β2 agonists cause activation of the adenylyl cyclase, not inhibition (A). Decreased leukotriene production (B) is the action of zileuton. Inactivation of NF-κB is the action of corticosteroids (D). Phosphodiesterase inhibition (E) is the action of methylxanthine.

A 64-year-old woman with a medical history of heart failure with reduced ejection fraction presents with new-onset shortness of breath. On exam, she appears to be in no distress and is able to speak in full sentences. She has bilateral 2+ pitting edema and venous stasis changes. An x-ray is ordered to evaluate her shortness of breath further. Interpretation of the film is suggestive of pleural effusion. By which mechanism did this patient's pleural effusion most likely develop? A. Decreased plasma oncotic pressure B. Increased intravascular pressure C. Increased renin-angiotensin aldosterone system (RAAS) activity D. Increased systemic vascular resistance E. Increased vascular permeability

The correct answer is increased intravascular pressure (B). This patient with heart failure presents with worsening shortness of breath in the setting of a pleural effusion. Remembering Starling forces, there are a number of ways in which a pleural effusion may develop. In this patient with reduced ejection fraction and signs of volume overload, it is likely she developed increased intravascular (hydrostatic) pressure, which lead to the formation of the fluid collection. There is no evidence of this patient having a pathologic state that would cause decreased plasma oncotic pressure (A). A change in RAAS activity (C) would not likely be the cause in this patient, nor would peripheral resistance (D) or vascular permeability (E).

Which of the following physical exam findings could be found in a patient with pulmonary hypertension? A. Diastolic decrescendo murmur B. Mid-systolic click C. Increased pulmonic component of second heart sound at left parasternal border D. Sustained apical impulse at apex E. Systolic ejection murmur at the right second intercostal space

The correct answer is increased pulmonic component of the second heart sound at left parasternal border (C). Pulmonary hypertension accentuates the pulmonary component of the second heart sound, which is best heard at the pulmonic area, at the left second intercostal space. Diastolic decrescendo murmur (A) is heard in patients with aortic regurgitation. Sustained apical impulse at apex (D) is seen in patients with left ventricular hypertrophy. Systolic ejection murmur at the right second intercostal space (E) is highly associated with aortic stenosis.

Which of the following best describes the mechanism of action of lumacaftor? A. CFTR potentiator that facilitates increased chloride transport by potentiating the channel-open probability of the CFTR protein at the cell surface B. Cleaves DNA to reduce the viscosity of mucus in the lungs C. Hydrates the airway mucus secretions and facilitates mucociliary function D. Improves the conformational stability of ΔF508-CFTR, resulting in increased processing and trafficking of mature protein to the cell surface E. Inhibits COX-1 and COX-2 to reduce prostaglandin formation and reduce inflammation in patients with CF

The correct answer is it improves the conformational stability of ΔF508-CFTR, resulting in increased processing and trafficking of mature protein to the cell surface (D). A CFTR potentiator that facilitates increased chloride transport by potentiating the channel-open probability of the CFTR protein at the cell surface (A) is the mechanism of action of ivacaftor, a similar drug. Cleaving DNA to reduce the viscosity of mucus in the lungs (B) is the mechanism of action of dornase α. Hydrating the airway mucus secretions and facilitating mucociliary function (C) is the mechanism of action of hypertonic saline. Inhibiting COX-1 and COX-2 to reduce prostaglandin formation and reduce inflammation in patients with CF (E) is the mechanism of action of ibuprofen, which is used as an anti-inflammatory agent. medication

All of the following are characteristics of chronic bronchitis except A. Goblet cell hyperplasia B. Involvement of the conducting zone C. Loss of distal airway elasticity D. Productive cough E. Squamous cell metaplasia

The correct answer is loss of distal airway elasticity (C). Of the choices given, only this response does not correlate with chronic bronchitis. Loss of elasticity of distal airways correlates with the disease process of emphysema. Goblet cell hyperplasia (A), involvement of the conducting zone (B), productive cough (D), and squamous cell metaplasia (E) are all features of chronic bronchitis.

Which of the causes of hypoxia does not have an increase in A-a O2 gradient? A. Impaired diffusion B. Low inspired O2tension C. Right-to-left shunt D. V/Q mismatch

The correct answer is low inspired O2 tension (B). A pulmonary problem that causes hypoxemia with a normal A-a O2 gradient signifies extrapulmonary causes. These are hypoventilation or low inspired O2 tension (B). Increased A-a O2 gradient occurs when there is an intrapulmonary cause for hypoxemia. Impaired diffusion (A), right-to-left shunt (C), and V/Q mismatch (D) are intrapulmonary causes of hypoxemia, which increase A-a O2 gradient.

Jack is a pilot who is currently flying a plane at high altitude. The plane's pressure system malfunctions, resulting in depressurization of the plane cabin. Jack's respiratory rate starts to increase from 12/min to 26/min. He also starts to experience signs of blackout. Assuming there is no air embolus forming in Jack's blood stream, what is the most likely cause of his symptoms? A. Hypoventilation B. Impaired diffusion C. Low inspired O2tension D. Right-to-left shunt E. V/Q mismatch

The correct answer is low inspired O2 tension (C). Due to the depressurization, Jack is currently breathing air at a lower atmospheric pressure. Even though there is still 21% O2 in the air, the PO2 is lowered as the total pressure is lowered. As a result, the alveoli only receive low O2 tension. Hypoventilation (A) is incorrect because Jack's respiratory rate has increased. Because Jack was functioning normally before the depressurization, there is less likely to be a pathological cause in Jack's lung, making an intrapulmonary cause of hypoxemia—impaired diffusion (B), right-to-left shunt (D) or V/Q mismatch (E)—unlikely.

Jack is a pilot who is currently flying a plane at high altitude. The plane's pressure system malfunctions, resulting in depressurization of the plane cabin. Jack's respiratory rate starts to increase from 12/min to 26/min. He also starts to experience signs of blackout. Assuming there is no air embolus forming in Jack's blood stream, what is the most likely cause of his symptoms? A. Hypoventilation B. Impaired diffusion C. Low inspired O2tension D. Right-to-left shunt E. V/Q mismatch

The correct answer is low inspired O2 tension (C). Due to the depressurization, Jack is currently breathing air at a lower atmospheric pressure. Even though there is still 21% O2 in the air, the PO2 is lowered as the total pressure is lowered. As a result, the alveoli only receive low O2 tension. Hypoventilation (A) is incorrect because Jack's respiratory rate has increased. Because Jack was functioning normally before the depressurization, there is less likely to be a pathological cause in Jack's lung, making an intrapulmonary cause of hypoxemia—impaired diffusion (B), right-to-left shunt (D) or V/Q mismatch (E)—unlikely.

What is the most common cause of cancer deaths in the United States? A. Breast B. Colon C. Kidney D. Lung E. Prostate

The correct answer is lung cancer (D). The most likely origin of cancer death in both men and women is the lung. Cancers of the breast (A) are the most commonly diagnosed cancers in women but do not represent the most cancer deaths. Colon cancer (B) is the third most commonly diagnosed tumor and cause of cancer death. Kidney cancer (C) is not common enough to account for a large proportion of cancer deaths. Cancers of the prostate (E) are the most commonly diagnosed cancers in men but are also less likely to cause death.

Which of the following is not a clinical exam used to assess for a pulmonary embolus or deep vein thrombosis? A. CT pulmonary angiography B. D-dimer C. Doppler ultrasound D. Echocardiography E. Magnetic resonance imaging (MRI)

The correct answer is magnetic resonance imaging (MRI) (E). This imaging technique is not useful in the assessment of pulmonary embolus and deep vein thrombosis because of low sensitivity and a high number of inconclusive results. D-dimer (B), Doppler ultrasound (C), CT pulmonary angiography (A), and echocardiography (D) are all diagnostic tools that can be used to screen for DVT and PEs.

Which is not a proven risk factor for developing lung cancer? A. Asbestos B. Marijuana use C. P450 monooxygenase polymorphisms D. Secondhand smoke E. Smoking cigarettes

The correct answer is marijuana use (B). Marijuana use has not been proven to be a risk factor for developing lung cancer. Asbestos (A) is an independent risk factor and substantially increases a cigarette smoker's risk. Many P450 monooxygenase polymorphisms (C) carry an increased risk for lung cancer because of the increased production of carcinogens from cigarette smoke. Secondhand smoke (D) can double one's risk. Smoking cigarettes (E) is the strongest risk factor for developing lung cancer.

Which of the following would be an example of pulmonary hypertension? A. Blood pressure of 140/90 mm Hg with asthma B. Blood pressure of 140/90 mm Hg with COPD C. Mean pulmonary arterial pressure = 20 mm Hg while jogging D. Mean pulmonary arterial pressure = 33 mm Hg while in bed E. Pulmonary capillary wedge pressure = 18 mm Hg

The correct answer is mean pulmonary arterial pressure = 33 mm Hg while in bed (D). Pulmonary hypertension is defined by a mean pulmonary arterial pressure ≥20 mm Hg when at rest. A blood pressure of 140/90 mm Hg with asthma or COPD (A and B) would be an example of systemic hypertension. In addition, right heart catheterization is used when the patient is in a resting position and not typically performed while undergoing strenuous activity, so a mean pulmonary arterial pressure of 20 mm Hg while jogging (C) is incorrect. Pulmonary capillary wedge pressure = 18 mm Hg (E) is incorrect; the pulmonary capillary wedge pressure is a measure of left atrial pressure, not mean pulmonary arterial pressure.

A patient with a history of poorly controlled epilepsy presents to the emergency room with fever, fatigue, and cough productive of putrid sputum. Chest x-ray confirms pulmonary abscess with air-fluid levels. Which antibiotic should be initiated? A. Aminoglycosides B. Meropenem C. Metronidazole D. Penicillin E. Vancomycin

The correct answer is meropenem (B). This patient likely has a lung abscess from aspiration of oropharyngeal secretions, and meropenem is a broad-spectrum antibiotic that can be used empirically for lung abscess. Aminoglycosides (A) are not used in the treatment of pulmonary abscesses due to poor penetration. Metronidazole (C) is not effective in the treatment of polymicrobial infections. Penicillin (D) is useful for treating Actinomyces. Vancomycin (E) is useful for treating MRSA.

A 67-year-old woman who has smoked for 25 years has been diagnosed with small cell lung cancer. Her physician recommends chemotherapy and insists that surgery will provide no benefit. What is the most likely reason surgery is not suggested for this patient? A. Bleeding complications B. Metastases C. Paraneoplastic syndrome D. Size of the tumor E. Too expensive

The correct answer is metastases (B). Small cell carcinoma is highly malignant and will most likely present with metastases. This reduces the benefit of surgery and makes chemotherapy the best option for treatment. Complications (A) may occur but such a risk would not prevent surgical intervention. Paraneoplastic syndromes (C) are often caused by small cell carcinoma but are less likely to preclude surgery. Although tumor size (D) and expense (E) do play roles in medical decision making, they are not as important as the presence of metastases.

Which of the following is a known risk factor for developing chronic bronchitis? A. Multiple infections B. Obesity C. Overexertion D. Previous lung surgery E. Recent antibiotic use

The correct answer is multiple infections (A). Of the choices, only multiple infections is associated with an increased risk of developing chronic bronchitis. Although obesity (B), overexertion (C), previous lung surgery (D), and recent antibiotic use (E) may be risk factors for other disease processes, they are not known to be associated with the development of chronic bronchitis.

Which of the following is the most common mutation of the CFTR gene in CF? A. F1074L B. G551D C. L206W D. ΔF508 E. S1251N

The correct answer is the ΔF508 mutation (D). The most common mutation of the CFTR gene in CF is the ΔF508 mutation. F1074L (A), G5551D (B), L206W (C), and S1251N (E) are all other mutations in the CFTR gene but are not the most common.

Two hours after ipratropium is given in the emergency department to a 25-year-old man, he begins having a headache, nausea, and dizziness. He also complains of being thirsty and asks the nurse to bring him water. What is the mechanism of action of the drug most likely responsible for these symptoms? A. β2receptor agonism B. Leukotriene production inhibition C. Leukotriene receptor blockade D. Mast cell stabilization E. Muscarinic receptor antagonism

The correct answer is muscarinic receptor antagonism (E). This patient is most likely experiencing anticholinergic symptoms, which can include dry mouth, blurry vision, headache, nausea, and drowsiness. These symptoms can sometimes be seen in patients taking muscarinic antagonists such as ipratropium and tiotropium. β2 receptor agonism (A) is the mechanism of action of β2 agonists, which does not cause the adverse effects seen in this patient. Montelukast and zafirlukast decrease asthma symptoms by inhibiting leukotriene production through blockage of the cysteinyl leukotriene receptor. These anti-leukotriene agents are generally well tolerated and are not associated with the adverse effects seen here (B and C). Mast cell stabilization (D) is the mechanism of action of cromolyn sodium, which can cause throat irritation but not the adverse effects described in this scenario.

Of the following options, which best describes the pathogenesis of Kartagener syndrome? A. Airway obstruction B. Mutation in theCFTRgene C. Mutation in dynein D. Recurrent respiratory infections E. Trauma to the lung

The correct answer is mutation in dynein (C). Kartagener syndrome is an inherited disorder in which the dynein protein is mutated, leading to immotile cilia. A mutation in the CFTR gene (B) is the pathogenesis of cystic fibrosis. Airway obstruction (A) and recurrent respiratory infections (D) both contribute to the development of bronchiectasis, which may occur as a consequence of Kartagener syndrome. Trauma to the lung (E) does not cause Kartagener syndrome, cystic fibrosis, or bronchiectasis.

Which of the following best describes the relationship of a pulmonary abscess to the lung parenchyma? A. Apoptosis B. Infarction C. Ischemia D. Necrosis E. Scarring

The correct answer is necrosis (D). A pulmonary abscess is defined as necrosis of the lung parenchyma, most often caused by bacterial infection. Apoptosis (A) is adenosine triphosphate (ATP)-dependent programmed cell death. Infarction (B) results from vascular occlusion and can be hemorrhagic in tissues with multiple blood supplies or pale in tissues with a single blood supply. Ischemia (C) occurs when there is an inadequate blood supply to meet demands and can lead to infarction. Scarring (E) results from deposition of granulation tissue and collagen during wound healing.

A doctor would like to determine the residual volume of a patient who has a decreased oxygen saturation. Which test would likely be the best choice for this patient? A. Arterial blood gases B. Bronchoprovocation test C. Nitrogen washout test D. Pulse oximetry E. Spirometry

The correct answer is nitrogen washout test (C). This test gives the patient 100% oxygen, which would likely help increase the oxygen saturation, in addition to determining the residual volume. Arterial blood gases (A) can determine gas partial pressures. Bronchoprovocation testing (B) is used to determine the presence of asthma. Pulse oximetry (D) determines hemoglobin saturation. Traditional spirometry (E) cannot measure residual volume.

Twenty-five minutes after the placement of a central line in the right internal jugular vein, a patient in the intensive care unit begins to experience shortness of breath. The central line was being placed for dialysis access, and the patient has no pulmonary history. On exam, there are bilateral breath sounds, and there is no hyperresonance on percussion. Concerned, the resident orders a chest radiograph. The image shows a small, apical pneumothorax. The pulse and blood pressure were the same before and after the procedure. What is the most appropriate next step in the management of this patient? A. Administration of 100% FiO2via tracheal intubation B. Diuresis C. Initiation of broad-spectrum antibiotics D. Observation E. Tube thoracostomy

The correct answer is observation (D). This patient experienced an iatrogenic pneumothorax after a misplaced central line. In the absence of clinical symptoms, a small pneumothorax can be observed because it will often resolve spontaneously. There is no indication to pursue invasive measures such as intubation (A) or thoracostomy (E). There is no indication that the patient is fluid overloaded, so diuresis is not an appropriate option in this patient (B). There is no indication to start antibiotics (C).

A 13-year-old boy comes to the emergency department for difficulty breathing, cough, and wheeze. He is given bronchodilator therapy as his condition is caused by bronchoconstriction. This condition is classified as what sort of disease? A. Arteritis B. Infectious C. Metastatic D. Obstructive E. Restrictive

The correct answer is obstructive lung disease (D). This child has bronchial asthma, a disease caused by bronchoconstriction. Asthma, bronchiectasis, chronic bronchitis, and emphysema are classified as obstructive. Restrictive lung diseases (E) include many conditions, including arteritis (A), infectious (B), and metastatic (C) diseases.

A 72-year-old retired foundry supervisor presents to your office with a 1-year history of worsening cough and shortness of breath; he appears tachypneic (abnormally rapid breathing). You diagnose him with silicosis after a detailed exam and chest x-ray. Which historical factor most likely led to the diagnosis of silicosis? A. Age B. Appearance C. Duration of symptoms D. Occupation E. Physical examination

The correct answer is occupation (D). Individuals who worked in foundries are at an increased risk of developing silicosis. The patient's age (A) does not aid in the diagnosis. His appearance (B) of being tachypneic is nonspecific and can be due to any pneumoconiosis or many other possibilities, such as pneumonia or heart failure. All of the pneumoconioses lead to chronic respiratory symptoms and would not help isolate the diagnosis (C). There are no physical exam findings specific for silicosis (E).

Which of these populations is considered at highest risk for getting adenovirus pneumonia? A. Athletes B. Military recruits C. Organ transplant recipients D. Recent travelers to the southwestern United States

The correct answer is organ transplant recipients (C). These patients are frequently severely immunosuppressed because of their medications to prevent transplant rejection. Athletes (A) and military recruits (B) are generally healthy adults who likely won't develop pneumonia from adenovirus. There is no relevant increase in adenovirus infection related to the Southwest, where instead coccidioidomycosis is a common regional illness (D).

A 9-year-old boy was brought to urgent care by his mother because 2 hours ago, he began having severe shortness of breath after playing in his aunt's grass field. On examination, the boy has some expiratory wheezes with prolonged expiration. You instructed the mother to give this boy a medicine prior to playing in grass in the future to prevent acute asthma attacks due to allergens. What is the likely mechanism of this medication? A. Blocks H2histamine receptor B. Blocks muscarinic receptor C. Inhibits lipoxygenase D. Prevents mast cell degranulation E. Stimulates β2adrenergic receptors

The correct answer is prevents mast cell degranulation (D). The drug described is cromolyn sodium, which stabilizes mast cells and inhibits the release of inflammatory mediators as well as histamine. H2 histamine receptor blockade (A) is the action of antihistamines such as cimetidine, famotidine, and ranitidine. They are used to treat gastroesophageal reflux disease and have limited role in asthma attacks. Blockade of muscarinic receptor (B) is the action of antimuscarinic agents such as ipratropium and tiotropium. Blockade of leukotriene receptor (by inhibiting lipoxygenase [C]) is the action of montelukast and zafirlukast. Stimulation of β2-adrenergic receptors is the action of both SABA and LABA (E).

What are the three most commonly diagnosed cancers in men in order from most to least common? A. Breast, lung, colorectal B. Colorectal, lung, prostate C. Lung, prostate, colorectal D. Prostate, lung, colorectal E. Prostate, lung, kidney

The correct answer is prostate, lung, colorectal (D). Lung cancer is the most likely cause of a cancer-related death but is the second most likely to be diagnosed. Prostate (19%) is the most commonly diagnosed cancer in men, followed by lung (14%) and colorectal (9%) (B, C). Breast cancer is the most commonly diagnosed cancer in women (30%) (A, E).

A 14-year-old child experiencing an acute asthma attack is treated with albuterol. This treatment results in bronchodilation of the airway and a two-fold increase in the diameter of the conducting system. Which of the following describes the resistance in the patient's airway after treatment as compared with before receiving albuterol? A. Resistance decreases by a factor of 4 B. Resistance decreases by a factor of 16 C. Resistance decreases by a factor of 32 D. Resistance increases by a factor of 4 E. Resistance increases by a factor of 16

The correct answer is resistance decreases by a factor of 16 (B). This can be derived from Poiseuille law (R = 8ηl/πr4). When the radius (r) is doubled in this patient, the resistance will be decreased by a factor of 16 (because 24 = 16). If the airway radius increased in size by a factor of 0.5, then the resistance decreases by a factor of 4 (A). If the radius increased by a factor of 5, then the resistance would decrease by a factor of 32 (C). If the size of the airway was decreased by a factor of 0.5, the resistance would increase by a factor of 4 (D). Decreasing the size of the airway by a factor of 2 would increase the resistance by a factor of 16 (E).

A patient with pulmonary sarcoidosis undergoes pulmonary function testing. Which of the following would most likely be seen? A. Increased functional residual capacity B. Increased residual volume C. Increased total lung capacity D. Obstructive pattern E. Restrictive pattern

The correct answer is restrictive pattern (E). Patients with pulmonary sarcoidosis often develop pulmonary fibrosis, which is characterized by a restrictive pattern on pulmonary function testing. Increased functional residual capacity (A), increased residual volume (B), and increased total lung capacity (C) are characteristic of obstructive lung disease. This patient would not be expected to have an obstructive pattern (D) as is observed in asthma and chronic obstructive pulmonary disease.

Mr. Smith is a 67-year-old man with medical history of chronic obstructive pulmonary disease. He comes to see you because of shortness of breath. Laboratory findings show low arterial PO2 and a widened A-a O2 gradient. When you give him 3 L of O2 via nasal cannula, his A-a O2 gradient does not improve. What is the most likely the cause of hypoxemia? A. Hypoventilation B. Impaired diffusion C. Low inspired O2tension D. Right-to-left shunt E. V/Q mismatch

The correct answer is right-to-left shunt (D). From his low arterial PO2, he has hypoxemia. Because of the widened A-a O2 gradient, he can have impaired diffusion, right-to-left shunt, or V/Q mismatch. Because diffusion defect (B) and V/Q mismatch (E) can be fixed by supplemental oxygen, but the right-to-left shunt cannot always be fixed by supplemental oxygen, the patient most likely has a right-to-left shunt. Both hypoventilation (A) and low inspired O2 (C) can cause hypoxemia and be corrected by supplemental oxygen, but they do not cause a high A-a O2 gradient.

Mr. Smith is a 67-year-old man with medical history of chronic obstructive pulmonary disease. He comes to see you because of shortness of breath. Laboratory findings show low arterial PO2 and a widened A-a O2 gradient. When you give him 3 L of O2 via nasal cannula, his A-a O2 gradient does not improve. What is the most likely the cause of hypoxemia? A. Hypoventilation B. Impaired diffusion C. Low inspired O2tension D. Right-to-left shunt E. V/Q mismatch

The correct answer is right-to-left shunt (D). From his low arterial PO2, he has hypoxemia. Because of the widened A-a O2 gradient, he can have impaired diffusion, right-to-left shunt, or V/Q mismatch. Because diffusion defect (B) and V/Q mismatch (E) can be fixed by supplemental oxygen, but the right-to-left shunt cannot always be fixed by supplemental oxygen, the patient most likely has a right-to-left shunt. Both hypoventilation (A) and low inspired O2 (C) can cause hypoxemia and be corrected by supplemental oxygen, but they do not cause a high A-a O2 gradient.

What animal is the reservoir for hantavirus? A. Armadillos B. Chickens C. Fleas D. Monkeys E. Rodents

The correct answer is rodents (E). Hantavirus is a single-stranded RNA virus that can cause severe pulmonary syndrome, resulting in death. Hantavirus is spread to humans from rodents. Armadillos are the reservoirs for Mycobacterium leprae, which causes leprosy (A). Fleas are the reservoirs for Yersinia pestis, which causes the plague (C). Chickens can carry avian influenza; this has not been seen in humans in the United States, but it has been present in other parts of the world (B). Monkeys and humans can act as reservoirs for yellow virus, which causes yellow fever (D).

Which of the following is not a histological feature of asthma? A. Charcot-Leyden crystals B. Curschmann spirals C. Mucus plugs D. Scarring of the lung parenchyma E. Thickening of the basement membrane and smooth muscle tissue.

The correct answer is scarring of the lung parenchyma (D). In asthma, the inflammation involves the peripheral airway and spares the lung parenchyma. Charcot-Leyden crystals are precipitated IgE proteins (A) and can be seen with asthma. Curschmann spirals (B) are mucus plugs (C), which can be seen in patients with asthma. Airway remodeling due to chronic inflammation in asthmatic patients can be seen as thickening of the basement membrane and smooth muscle tissue (E).

A 59-year-old man complains of productive cough with blood, fever, night sweats, and weight loss. He has no medical history except for one hospitalization 20 years ago for a severe lung infection. Social history reveals that he was incarcerated at that time. Chest x-ray reveals upper lobe cavitation and mediastinal lymphadenopathy. Sputum cultures are positive for Mycobacterium tuberculosis. What type of tuberculosis infection does this patient have? A. Latent tuberculosis B. Miliary tuberculosis C. Pott disease D. Primary active tuberculosis E. Secondary tuberculosis

The correct answer is secondary tuberculosis (E). The patient has a history of previous tuberculosis infection, which explains the upper lung cavitations seen on chest x-ray. Latent tuberculosis (A) is incorrect because this patient has active infection as evidenced by his symptoms and sputum cultures. Miliary tuberculosis (B) is incorrect because his chest radiograph does not reveal a miliary pattern and he has no risk factors for disseminated disease. Pott disease (C) is the involvement of tuberculosis infection in the vertebral bodies, which this patient does not likely have as he does not complain of back pain. Primary active tuberculosis (D) is also incorrect because the patient has had infection in the past, as evidenced by the cavities in the upper lobes of the lung.

A 23-year-old woman is brought to the emergency department confused. Her exam is notable for a respiratory rate of 8/min. Her heart and lung exams are normal. Her arterial blood gas shows the following: pH 7.30 pCO2 55 mm Hg HCO3 26 mEq/L pO2 85 mm Hg Which of the following diagnoses is most likely? A. Asthma B. Myasthenia gravis C. Obstructive sleep apnea D. Pneumococcal pneumonia E. Sedative overdose

The correct answer is sedative overdose (E). The patient has a respiratory acidosis with low pH and elevated pCO2. Asthma (A) would only cause respiratory acidosis if severe or the patient is tiring, and there is no wheezing described in this patient. Myasthenia gravis (B) causes respiratory acidosis from respiratory muscle weakness, but the CNS (central) drive is intact, so there would be a high respiratory rate, not low. Sleep apnea (C) can cause respiratory acidosis but is seen mostly in older obese patients. Pneumonia (D) causes hypoxia that stimulates hyperventilation, so it would cause a respiratory alkalosis.

Which treatment of chronic bronchitis listed below is the only one to lower mortality? A. Albuterol B. Ipratropium bromide C. Salmeterol D. Smoking cessation E. Tiotropium

The correct answer is smoking cessation (D). Although albuterol (A), ipratropium bromide (B), salmeterol (C), and tiotropium (D) are all treatments for chronic bronchitis, they are only useful for symptomatic control. Only smoking cessation reduces mortality (by 18%).

Which pathological change with a BMPR2 mutation results in pulmonary arterial hypertension? A. Fibrosis B. Hypoxic vasoconstriction C. Mechanical obstruction D. Pulmonary congestion E. Smooth muscle proliferation

The correct answer is smooth muscle proliferation (E). Pulmonary arterial hypertension relates to intrinsic causes of vessel change. In pulmonary arterial hypertension, the mutation of the BMPR2 gene leads to unregulated growth from growth factors and results in smooth muscle proliferation. The increased thickness of the vessel wall causes an increased resistance of the wall, and thus pulmonary arterial hypertension. Fibrosis (A), hypoxic vasoconstriction (B), mechanical obstruction (C), and pulmonary congestion (D) are all extrinsic causes of pulmonary hypertension.

A smoker of 40 years is found to have a centrally located lung tumor that is observed to have collections of keratin, intercellular bridges, and high mitotic activity. What is the most likely histologic classification of the tumor? A. Adenocarcinoma B. Large cell carcinoma C. Lymphoma D. Small cell carcinoma E. Squamous cell carcinoma

The correct answer is squamous cell carcinoma (E). Because squamous cell carcinoma originates from bronchial epithelium, it is often found centrally. The squamous cells will often form intercellular bridges and keratin pearls. Adenocarcinoma (A) is usually found peripherally and is made of glandular epithelium, producing mucin. The presence of keratin pearls is not associated with large cell carcinoma (B), lymphoma (C), or small cell carcinoma (D).

Which of the following symptoms is the classic presentation of a patient experiencing an atypical presentation of pneumonia? A. Acute onset illness B. Dyspnea C. High fevers D. Severe productive cough E. Subacute onset illness

The correct answer is subacute onset illness (E). Atypical presentations tend to be more indolent and slow in onset with much more mild symptoms as compared with typical pneumonia. Acute onset illness (A), dyspnea (B), high fevers (C), and severe productive cough (D) are more classically seen in typical pneumonia.

A 35-year-old man who recently immigrated from Iran has arrived to your urgent care clinic complaining of sudden onset fever, chills, and cough with shortness of breath. He just arrived from his home country several days prior and states that his family members had similar symptoms several days ago and were since hospitalized. You diagnose this patient with suspected MERS. What is the best treatment for his condition? A. Amantadine B. Antibiotics C. Glucocorticoids D. Ribavirin E. Supportive care

The correct answer is supportive care (E). There is currently no standard effective treatment for SARS or MERS. Although many clinicians will use glucocorticoids (C) and ribavirin (D), there has been no evidence to show these treatments actually work. Amantadine (A) is used to shorten the duration of influenza illness, and antibiotics (B) are only helpful for bacterial infections.

A 45-year-old African-American woman presents to the clinic for a 3-month history of dry cough and increasing shortness of breath, especially on exertion. She is not hypoxemic at rest or with exertion. She has a background history of sarcoidosis. She is seeking treatment because these symptoms are negatively affecting her quality of life. What is the best treatment option to control her respiratory symptoms? A. Albuterol B. Inhaled glucocorticoids C. Long-term oxygen therapy D. Salmeterol E. Systemic glucocorticoids

The correct answer is systemic glucocorticoids (E). This patient most likely developed pulmonary sarcoidosis. Systemic glucocorticoids are most helpful for her condition because the disease is immune mediated. Albuterol and salmeterol are bronchodilators, which can be used for obstructive lung disease such as asthma. Albuterol (A) and salmeterol (D) are unlikely to be helpful in pulmonary sarcoidosis because it causes a restrictive lung disease. Inhaled glucocorticoids (B) have been attempted in experimental settings but were not successful in controlling the symptoms of pulmonary sarcoidosis. Long-term oxygen therapy (C) is unlikely to control the symptoms because it does not alter or halt the disease process causing pulmonary sarcoidosis.

A 70-year-old man presents to the emergency department with breathing difficulty. He admits to smoking two packs of cigarettes a day for the past 40 years. After extensive testing, he is diagnosed with emphysema. Of the following sets of parameters, which would be characteristic of this patient's respiration? A. ↑ compliance, ↑ resistance, ↑ turbulent flow B. ↑ compliance, ↑ resistance, ↓ turbulent flow C. ↑ compliance, ↓ resistance, ↓ turbulent flow D. ↓ compliance, ↓ resistance, ↓ turbulent flow

The correct answer is ↑ compliance, ↑ resistance, ↑ turbulent flow (A). Emphysema is characterized by air trapping, increased airway resistance, and increased lung volumes. As a result, this patient's lungs would demonstrate increased compliance, increased turbulent flow, and increased overall resistance. Answer (B) is incorrect because it specifies decreased turbulent flow instead. An increase in lung compliance would cause an increase in airway resistance, not a decrease in resistance (C). Decreases in compliance, resistance, and turbulent flow describe the pathology seen in restrictive lung disease (ie, pulmonary fibrosis), in which there is a marked reduction in the ability to fill the lungs (D).


संबंधित स्टडी सेट्स

ACTUAL FRENCH FINAL 10th grade chan

View Set

Chapter 4 - Life Insurance Riders

View Set

Preanesthetic period & anesthetic agents

View Set

Chapter 14: Ways to build Confidence

View Set

Parasitic worms (Helminths) (Ch.12)

View Set

Les nationalités, les pays d'origine, le verbe être et le verbe venir

View Set

Lesson 9 - Estructura 9.1 - Lesson-Check ALL - Irregular preterites,

View Set